A quick question about complex integrals and Cauchy's integral formula












4












$begingroup$


I'll spare the specifics for brevity's sake, but in essence the problem I'm posed is finding



$$int_C frac{(z-1)^3 cdot e^z cdot cos(z)}{z}dz$$



along two different closed loops $C$. Each is a rectangle, oriented clockwise. One of them encloses the discontinuity of this function (i.e. $z=0$) and another doesn't.




Post-Script (December 2018): I recognize that the "discontinuity" mentioned is in reality a singularity.




I'm mostly just wanting to double-check my approach to this since it's explicitly specified that "this shouldn't take much computation," and want to double-check I'm on the right path.



My thoughts on the matter:




  • For $C$ being the closed loop not enclosing the discontinuity, the integral would be $0$ per Cauchy's integral theorem.


  • For $C$ being the closed loop that encloses the discontinuity, the integral would be $2pi i f(0)$, from Cauchy's integral formula, where $f(z)$ is the numerator of the integrand (sans the $dz$ of course), and "$0$" coming from being the point of discontinuity.



I have a rough intuition for why this might be - it's fairly heuristic and informal though - so I just wanted to make sure I was on the right track.



Thanks in advance.










share|cite|improve this question











$endgroup$








  • 1




    $begingroup$
    You are right and the value of the integral in the second case is $-1$.
    $endgroup$
    – Kavi Rama Murthy
    Oct 12 '18 at 7:50










  • $begingroup$
    You have already been told that you are right. I just want to add that the function that you are integrating has no point of discontinuity. The point $0$ is a singularity.
    $endgroup$
    – José Carlos Santos
    Oct 12 '18 at 8:00






  • 1




    $begingroup$
    Actually @KaviRamaMurthy I did have a second thought: the orientation of the curve. Since it's clockwise, and the formulation of Cauchy's formula states counterclockwise, would it not be the negative of that? (Another minor nitpick but I think it should be noted $f(0)=-1)$ as opposed to the integral which is $2pi i f(0)$. Not trying to be a jerk though.)
    $endgroup$
    – Eevee Trainer
    Oct 12 '18 at 8:01






  • 2




    $begingroup$
    @EeveeTrainer The correct answer is $2pi i$. I was a bit careless in my earlier comment.
    $endgroup$
    – Kavi Rama Murthy
    Oct 12 '18 at 8:15






  • 1




    $begingroup$
    @EeveeTrainer A function can only be continuous (or discontinuous) at points of its domain. An isolated singularity is a point outside the domain such that all points nearby belong to it.
    $endgroup$
    – José Carlos Santos
    Oct 12 '18 at 8:17
















4












$begingroup$


I'll spare the specifics for brevity's sake, but in essence the problem I'm posed is finding



$$int_C frac{(z-1)^3 cdot e^z cdot cos(z)}{z}dz$$



along two different closed loops $C$. Each is a rectangle, oriented clockwise. One of them encloses the discontinuity of this function (i.e. $z=0$) and another doesn't.




Post-Script (December 2018): I recognize that the "discontinuity" mentioned is in reality a singularity.




I'm mostly just wanting to double-check my approach to this since it's explicitly specified that "this shouldn't take much computation," and want to double-check I'm on the right path.



My thoughts on the matter:




  • For $C$ being the closed loop not enclosing the discontinuity, the integral would be $0$ per Cauchy's integral theorem.


  • For $C$ being the closed loop that encloses the discontinuity, the integral would be $2pi i f(0)$, from Cauchy's integral formula, where $f(z)$ is the numerator of the integrand (sans the $dz$ of course), and "$0$" coming from being the point of discontinuity.



I have a rough intuition for why this might be - it's fairly heuristic and informal though - so I just wanted to make sure I was on the right track.



Thanks in advance.










share|cite|improve this question











$endgroup$








  • 1




    $begingroup$
    You are right and the value of the integral in the second case is $-1$.
    $endgroup$
    – Kavi Rama Murthy
    Oct 12 '18 at 7:50










  • $begingroup$
    You have already been told that you are right. I just want to add that the function that you are integrating has no point of discontinuity. The point $0$ is a singularity.
    $endgroup$
    – José Carlos Santos
    Oct 12 '18 at 8:00






  • 1




    $begingroup$
    Actually @KaviRamaMurthy I did have a second thought: the orientation of the curve. Since it's clockwise, and the formulation of Cauchy's formula states counterclockwise, would it not be the negative of that? (Another minor nitpick but I think it should be noted $f(0)=-1)$ as opposed to the integral which is $2pi i f(0)$. Not trying to be a jerk though.)
    $endgroup$
    – Eevee Trainer
    Oct 12 '18 at 8:01






  • 2




    $begingroup$
    @EeveeTrainer The correct answer is $2pi i$. I was a bit careless in my earlier comment.
    $endgroup$
    – Kavi Rama Murthy
    Oct 12 '18 at 8:15






  • 1




    $begingroup$
    @EeveeTrainer A function can only be continuous (or discontinuous) at points of its domain. An isolated singularity is a point outside the domain such that all points nearby belong to it.
    $endgroup$
    – José Carlos Santos
    Oct 12 '18 at 8:17














4












4








4





$begingroup$


I'll spare the specifics for brevity's sake, but in essence the problem I'm posed is finding



$$int_C frac{(z-1)^3 cdot e^z cdot cos(z)}{z}dz$$



along two different closed loops $C$. Each is a rectangle, oriented clockwise. One of them encloses the discontinuity of this function (i.e. $z=0$) and another doesn't.




Post-Script (December 2018): I recognize that the "discontinuity" mentioned is in reality a singularity.




I'm mostly just wanting to double-check my approach to this since it's explicitly specified that "this shouldn't take much computation," and want to double-check I'm on the right path.



My thoughts on the matter:




  • For $C$ being the closed loop not enclosing the discontinuity, the integral would be $0$ per Cauchy's integral theorem.


  • For $C$ being the closed loop that encloses the discontinuity, the integral would be $2pi i f(0)$, from Cauchy's integral formula, where $f(z)$ is the numerator of the integrand (sans the $dz$ of course), and "$0$" coming from being the point of discontinuity.



I have a rough intuition for why this might be - it's fairly heuristic and informal though - so I just wanted to make sure I was on the right track.



Thanks in advance.










share|cite|improve this question











$endgroup$




I'll spare the specifics for brevity's sake, but in essence the problem I'm posed is finding



$$int_C frac{(z-1)^3 cdot e^z cdot cos(z)}{z}dz$$



along two different closed loops $C$. Each is a rectangle, oriented clockwise. One of them encloses the discontinuity of this function (i.e. $z=0$) and another doesn't.




Post-Script (December 2018): I recognize that the "discontinuity" mentioned is in reality a singularity.




I'm mostly just wanting to double-check my approach to this since it's explicitly specified that "this shouldn't take much computation," and want to double-check I'm on the right path.



My thoughts on the matter:




  • For $C$ being the closed loop not enclosing the discontinuity, the integral would be $0$ per Cauchy's integral theorem.


  • For $C$ being the closed loop that encloses the discontinuity, the integral would be $2pi i f(0)$, from Cauchy's integral formula, where $f(z)$ is the numerator of the integrand (sans the $dz$ of course), and "$0$" coming from being the point of discontinuity.



I have a rough intuition for why this might be - it's fairly heuristic and informal though - so I just wanted to make sure I was on the right track.



Thanks in advance.







complex-analysis complex-integration






share|cite|improve this question















share|cite|improve this question













share|cite|improve this question




share|cite|improve this question








edited Dec 30 '18 at 8:43







Eevee Trainer

















asked Oct 12 '18 at 7:48









Eevee TrainerEevee Trainer

8,38921439




8,38921439








  • 1




    $begingroup$
    You are right and the value of the integral in the second case is $-1$.
    $endgroup$
    – Kavi Rama Murthy
    Oct 12 '18 at 7:50










  • $begingroup$
    You have already been told that you are right. I just want to add that the function that you are integrating has no point of discontinuity. The point $0$ is a singularity.
    $endgroup$
    – José Carlos Santos
    Oct 12 '18 at 8:00






  • 1




    $begingroup$
    Actually @KaviRamaMurthy I did have a second thought: the orientation of the curve. Since it's clockwise, and the formulation of Cauchy's formula states counterclockwise, would it not be the negative of that? (Another minor nitpick but I think it should be noted $f(0)=-1)$ as opposed to the integral which is $2pi i f(0)$. Not trying to be a jerk though.)
    $endgroup$
    – Eevee Trainer
    Oct 12 '18 at 8:01






  • 2




    $begingroup$
    @EeveeTrainer The correct answer is $2pi i$. I was a bit careless in my earlier comment.
    $endgroup$
    – Kavi Rama Murthy
    Oct 12 '18 at 8:15






  • 1




    $begingroup$
    @EeveeTrainer A function can only be continuous (or discontinuous) at points of its domain. An isolated singularity is a point outside the domain such that all points nearby belong to it.
    $endgroup$
    – José Carlos Santos
    Oct 12 '18 at 8:17














  • 1




    $begingroup$
    You are right and the value of the integral in the second case is $-1$.
    $endgroup$
    – Kavi Rama Murthy
    Oct 12 '18 at 7:50










  • $begingroup$
    You have already been told that you are right. I just want to add that the function that you are integrating has no point of discontinuity. The point $0$ is a singularity.
    $endgroup$
    – José Carlos Santos
    Oct 12 '18 at 8:00






  • 1




    $begingroup$
    Actually @KaviRamaMurthy I did have a second thought: the orientation of the curve. Since it's clockwise, and the formulation of Cauchy's formula states counterclockwise, would it not be the negative of that? (Another minor nitpick but I think it should be noted $f(0)=-1)$ as opposed to the integral which is $2pi i f(0)$. Not trying to be a jerk though.)
    $endgroup$
    – Eevee Trainer
    Oct 12 '18 at 8:01






  • 2




    $begingroup$
    @EeveeTrainer The correct answer is $2pi i$. I was a bit careless in my earlier comment.
    $endgroup$
    – Kavi Rama Murthy
    Oct 12 '18 at 8:15






  • 1




    $begingroup$
    @EeveeTrainer A function can only be continuous (or discontinuous) at points of its domain. An isolated singularity is a point outside the domain such that all points nearby belong to it.
    $endgroup$
    – José Carlos Santos
    Oct 12 '18 at 8:17








1




1




$begingroup$
You are right and the value of the integral in the second case is $-1$.
$endgroup$
– Kavi Rama Murthy
Oct 12 '18 at 7:50




$begingroup$
You are right and the value of the integral in the second case is $-1$.
$endgroup$
– Kavi Rama Murthy
Oct 12 '18 at 7:50












$begingroup$
You have already been told that you are right. I just want to add that the function that you are integrating has no point of discontinuity. The point $0$ is a singularity.
$endgroup$
– José Carlos Santos
Oct 12 '18 at 8:00




$begingroup$
You have already been told that you are right. I just want to add that the function that you are integrating has no point of discontinuity. The point $0$ is a singularity.
$endgroup$
– José Carlos Santos
Oct 12 '18 at 8:00




1




1




$begingroup$
Actually @KaviRamaMurthy I did have a second thought: the orientation of the curve. Since it's clockwise, and the formulation of Cauchy's formula states counterclockwise, would it not be the negative of that? (Another minor nitpick but I think it should be noted $f(0)=-1)$ as opposed to the integral which is $2pi i f(0)$. Not trying to be a jerk though.)
$endgroup$
– Eevee Trainer
Oct 12 '18 at 8:01




$begingroup$
Actually @KaviRamaMurthy I did have a second thought: the orientation of the curve. Since it's clockwise, and the formulation of Cauchy's formula states counterclockwise, would it not be the negative of that? (Another minor nitpick but I think it should be noted $f(0)=-1)$ as opposed to the integral which is $2pi i f(0)$. Not trying to be a jerk though.)
$endgroup$
– Eevee Trainer
Oct 12 '18 at 8:01




2




2




$begingroup$
@EeveeTrainer The correct answer is $2pi i$. I was a bit careless in my earlier comment.
$endgroup$
– Kavi Rama Murthy
Oct 12 '18 at 8:15




$begingroup$
@EeveeTrainer The correct answer is $2pi i$. I was a bit careless in my earlier comment.
$endgroup$
– Kavi Rama Murthy
Oct 12 '18 at 8:15




1




1




$begingroup$
@EeveeTrainer A function can only be continuous (or discontinuous) at points of its domain. An isolated singularity is a point outside the domain such that all points nearby belong to it.
$endgroup$
– José Carlos Santos
Oct 12 '18 at 8:17




$begingroup$
@EeveeTrainer A function can only be continuous (or discontinuous) at points of its domain. An isolated singularity is a point outside the domain such that all points nearby belong to it.
$endgroup$
– José Carlos Santos
Oct 12 '18 at 8:17










1 Answer
1






active

oldest

votes


















1












$begingroup$


Full disclosure: I was going back through some of the questions I asked over the past semester and noticed this one was left unanswered, at least in the "official answer" sense, i.e. no one gave me an answer to accept. Since this was basically an "am I right" sort of question, I'm just going to make this a community wiki answer and accept it for the sake of closing the question. (I chose community wiki because it was other people who verified my solution and I don't have much to add to the solution. Plus if others have discussion to offer, they should do so.)






The thoughts posted in the OP are indeed correct, and follow directly from Cauchy's integral theorem/formula. The first contour yields an integral evaluating to $0$, the second to $2pi i$.



Thanks to José Carlos Santos and Kavi Rama Murthy for their input on the post.






share|cite|improve this answer











$endgroup$













    Your Answer





    StackExchange.ifUsing("editor", function () {
    return StackExchange.using("mathjaxEditing", function () {
    StackExchange.MarkdownEditor.creationCallbacks.add(function (editor, postfix) {
    StackExchange.mathjaxEditing.prepareWmdForMathJax(editor, postfix, [["$", "$"], ["\\(","\\)"]]);
    });
    });
    }, "mathjax-editing");

    StackExchange.ready(function() {
    var channelOptions = {
    tags: "".split(" "),
    id: "69"
    };
    initTagRenderer("".split(" "), "".split(" "), channelOptions);

    StackExchange.using("externalEditor", function() {
    // Have to fire editor after snippets, if snippets enabled
    if (StackExchange.settings.snippets.snippetsEnabled) {
    StackExchange.using("snippets", function() {
    createEditor();
    });
    }
    else {
    createEditor();
    }
    });

    function createEditor() {
    StackExchange.prepareEditor({
    heartbeatType: 'answer',
    autoActivateHeartbeat: false,
    convertImagesToLinks: true,
    noModals: true,
    showLowRepImageUploadWarning: true,
    reputationToPostImages: 10,
    bindNavPrevention: true,
    postfix: "",
    imageUploader: {
    brandingHtml: "Powered by u003ca class="icon-imgur-white" href="https://imgur.com/"u003eu003c/au003e",
    contentPolicyHtml: "User contributions licensed under u003ca href="https://creativecommons.org/licenses/by-sa/3.0/"u003ecc by-sa 3.0 with attribution requiredu003c/au003e u003ca href="https://stackoverflow.com/legal/content-policy"u003e(content policy)u003c/au003e",
    allowUrls: true
    },
    noCode: true, onDemand: true,
    discardSelector: ".discard-answer"
    ,immediatelyShowMarkdownHelp:true
    });


    }
    });














    draft saved

    draft discarded


















    StackExchange.ready(
    function () {
    StackExchange.openid.initPostLogin('.new-post-login', 'https%3a%2f%2fmath.stackexchange.com%2fquestions%2f2952395%2fa-quick-question-about-complex-integrals-and-cauchys-integral-formula%23new-answer', 'question_page');
    }
    );

    Post as a guest















    Required, but never shown

























    1 Answer
    1






    active

    oldest

    votes








    1 Answer
    1






    active

    oldest

    votes









    active

    oldest

    votes






    active

    oldest

    votes









    1












    $begingroup$


    Full disclosure: I was going back through some of the questions I asked over the past semester and noticed this one was left unanswered, at least in the "official answer" sense, i.e. no one gave me an answer to accept. Since this was basically an "am I right" sort of question, I'm just going to make this a community wiki answer and accept it for the sake of closing the question. (I chose community wiki because it was other people who verified my solution and I don't have much to add to the solution. Plus if others have discussion to offer, they should do so.)






    The thoughts posted in the OP are indeed correct, and follow directly from Cauchy's integral theorem/formula. The first contour yields an integral evaluating to $0$, the second to $2pi i$.



    Thanks to José Carlos Santos and Kavi Rama Murthy for their input on the post.






    share|cite|improve this answer











    $endgroup$


















      1












      $begingroup$


      Full disclosure: I was going back through some of the questions I asked over the past semester and noticed this one was left unanswered, at least in the "official answer" sense, i.e. no one gave me an answer to accept. Since this was basically an "am I right" sort of question, I'm just going to make this a community wiki answer and accept it for the sake of closing the question. (I chose community wiki because it was other people who verified my solution and I don't have much to add to the solution. Plus if others have discussion to offer, they should do so.)






      The thoughts posted in the OP are indeed correct, and follow directly from Cauchy's integral theorem/formula. The first contour yields an integral evaluating to $0$, the second to $2pi i$.



      Thanks to José Carlos Santos and Kavi Rama Murthy for their input on the post.






      share|cite|improve this answer











      $endgroup$
















        1












        1








        1





        $begingroup$


        Full disclosure: I was going back through some of the questions I asked over the past semester and noticed this one was left unanswered, at least in the "official answer" sense, i.e. no one gave me an answer to accept. Since this was basically an "am I right" sort of question, I'm just going to make this a community wiki answer and accept it for the sake of closing the question. (I chose community wiki because it was other people who verified my solution and I don't have much to add to the solution. Plus if others have discussion to offer, they should do so.)






        The thoughts posted in the OP are indeed correct, and follow directly from Cauchy's integral theorem/formula. The first contour yields an integral evaluating to $0$, the second to $2pi i$.



        Thanks to José Carlos Santos and Kavi Rama Murthy for their input on the post.






        share|cite|improve this answer











        $endgroup$




        Full disclosure: I was going back through some of the questions I asked over the past semester and noticed this one was left unanswered, at least in the "official answer" sense, i.e. no one gave me an answer to accept. Since this was basically an "am I right" sort of question, I'm just going to make this a community wiki answer and accept it for the sake of closing the question. (I chose community wiki because it was other people who verified my solution and I don't have much to add to the solution. Plus if others have discussion to offer, they should do so.)






        The thoughts posted in the OP are indeed correct, and follow directly from Cauchy's integral theorem/formula. The first contour yields an integral evaluating to $0$, the second to $2pi i$.



        Thanks to José Carlos Santos and Kavi Rama Murthy for their input on the post.







        share|cite|improve this answer














        share|cite|improve this answer



        share|cite|improve this answer








        answered Dec 30 '18 at 8:34


























        community wiki





        Eevee Trainer































            draft saved

            draft discarded




















































            Thanks for contributing an answer to Mathematics Stack Exchange!


            • Please be sure to answer the question. Provide details and share your research!

            But avoid



            • Asking for help, clarification, or responding to other answers.

            • Making statements based on opinion; back them up with references or personal experience.


            Use MathJax to format equations. MathJax reference.


            To learn more, see our tips on writing great answers.




            draft saved


            draft discarded














            StackExchange.ready(
            function () {
            StackExchange.openid.initPostLogin('.new-post-login', 'https%3a%2f%2fmath.stackexchange.com%2fquestions%2f2952395%2fa-quick-question-about-complex-integrals-and-cauchys-integral-formula%23new-answer', 'question_page');
            }
            );

            Post as a guest















            Required, but never shown





















































            Required, but never shown














            Required, but never shown












            Required, but never shown







            Required, but never shown

































            Required, but never shown














            Required, but never shown












            Required, but never shown







            Required, but never shown







            Popular posts from this blog

            To store a contact into the json file from server.js file using a class in NodeJS

            Redirect URL with Chrome Remote Debugging Android Devices

            Dieringhausen